I scored a 169 on November. I'm thinking it had something to do with RC. I average a low 170. I want to take it up to the mid 170s. How do I do this for January. Please help :)
- Joined
- Jun 2025
- Subscription
- Live
i nearly got tripped up by this question. when i was like 12 i wanted to be an astronaut. ik they did weight bearing exercises on the ISS.Â
But B was so bad that I picked B. Thank God lol.Â
i was so scared to pick A because sample size ACs are basically never right lol
i got this question right and i still think its bullshit
my problem with this, even though I got it right, is "only some", because that implies we can predict the destination, but that isn't true according to the information we receive in the passage.
I think my core frustration with this question is that it trades on my traditional understanding of necessary assumption. So often we are used to seeing a necessary assumption which entails from the conclusion. Negating it would destroy the conclusion. We don't really need to think about the support because we get close to the realm of sufficient assumption. But in this question A sort of does that. I don't know if this makes sense. But I think it was because of this I was so slow in answering this question and has so many reservations about A.
The truth of A in a vacuum actually doesn't matter if A is true or not. The LSAT almost always preys on NA's for which this is the case.
my main gripe with this question is that the question asks us to weaken the claim that large size is a better defense than armor, but everyone treats the conclusion as if it were casual claim for some reason. I'm positive, if I was able to identify this question as casual in nature, B would've been an instant pick. But I didn't.
I get B. For sure, it's better than everything else. But it requires a number of assumptions. Principally that the argument is structured such that the author believes that this greater defensive feature of no armor is the reason why the fish doesn't have armor in the first place. Almost like they evolved this feature on defense alone.
This is a crazy assumption, but it opens the door to the alternative hypothesis. Because then, obviously, the actual reason why they don't have armor is just because they couldn't survive cold winters without them (duh).Â
But as it stands, it could be true that both B and the conclusion are true. They don't seem to weaken one another.
So what if size is an important factor for surviving cold winters? Does that somehow mean that no armor is not actually better? B merely indicates that larger size has another function.Â
If we strictly read the conclusion as saying: better defense from no armor. Then yeah B is kind of dogshit, it doesn't really do anything. But add this giga assumption of causality that every goddamn lsat place is applying (from where they all defaulted on this idk), then yeah B looks pretty good. Because now we have another reason why the fish could've wanted to get bigger.Â
"The author thinks the larger size must have something to do with protection from predators. But it might instead be due to the need for surviving cold winters."
Do you see the subtle shift in language in the written explanation? Whereas the actual conclusion is a dry, large size=better defense, the written explanation tries to inject a better defense-evolution hypothesis. There making a comparative judgement on defense. Not a casual claim on the emergence of a property.
#help Â
EDIT: I don't know if this is going to help anyone but I think I figured this out:
Armor slows growth.
Lake fish lack armor (so they can grow faster/bigger).
Therefore: in lakes, larger size is a better defense against predators than armor.
This argument evolves sort of inorganically. The premises don’t really support the conclusion. We don’t have any reason to believe that the larger sizer has anything to do with defense against predators. It's really random right? We need something that closes this gap between size and armor to comparative claims about defense. This is an assumption that takes place that we can target. From here we can get B.
I don't have a better explanation. Sorry.
you don't actually know whether it significantly reduces the oxygen content. That is a FAT assumption. A is way better.
I really feel like this is a weak question.
It seems very reasonable to assume that SOME development in the US has occurred. Just because it's fully established doesn't mean no future development can occur. It seems bizarre to place this prescription for this question.
Moreover, it certainly seems more reasonable than assuming that Maxines opinion on talk story is reflective of talk story in general.
I need help. Cause otherwise this question is capital B Bullshit.
#help
Why is B wrong? The final statement literally says future progress requires cooperation.
“historical sociologist Philip Abrams argues that, while people are made by society as much as society is made by people, sociologists' approach to the subject is usually to focus on only one of these forms of influence to the exclusion of the other.”Â
Does this statement imply that the sociologists are willingly choosing to ignore specific issues? It seems that the sociologists are missing out on specific perspectives by choosing a specific analysis mode. But that doesn’t meant they chose to miss out on those perspectives. Merely that there decisions lead them to missing out.Â
Just because someone makes a specific decision, doesn’t mean they acknowledge and willingly accept it’s consequences. How many times have I drank really hot coffee with no intention of burning my mouth. Did I deliberately ignore the fact that I would get burned? Maybe I was ignorant to the idea that I would get burned, perchance. But I don’t think it means that while drinking the coffee I was cognizant to the fact I would get burned and simply chose to ignore it’s consequences. This is what this question trades on.
So this is a bullshit question but let’s try to make sense of it. Note this question is an According to the passage type question and than note paragraph 3 : Modern critics of Victorian philanthropy often use the words "amateurish" or "inadequate" to describe Victorian philanthropy, as though Victorian charity can only be understood as an antecedent to the era of state-sponsored, professionally administered charity.Â
This quote essentially argues that both critiques view a specific inevitability to a professionally administered charity. So the author of the passage is saying to us, regardless of whether its true, that both think government charity is inevitable. This lines up well with D.Â
Unfortunately this isn’t satisfying because we want more specific evidence that the self-serving exercise critique assumes that it’s inevitable. But we don’t get it. Which is why this question is bullshit.
I understand this question and I get why D is right. But it's really evil.
I don't have the time to go through questions like this in the first place but to add a fake out with the first sentence in the correct AC. I eliminated D because I just didn't bother reading it entirely.
Level 5s like this happen with such infrequency that I think I'll be fine. But still, cringe question.
#admin #help
I don't really like C. I get why the rest are wrong and I did get this question using POE. But I still don't really like it.
First the conclusion is that recruitment is flatly dependent on dropouts. Even if the percentage of graduate recruits rose, it doesn't imply that the military is no longer dependent on those dropouts.
Let's use another example. Suppose there were 2 wells where water was being drawn from. G and F. Suppose that the total water drawn increased. Say that we know that the water supplied by G rose sharply. Can we conclude or even imply that means the water supply is less likely to be dependent on well F? How?
There is 1 way: if the rise in water was SO sharp that G now fulfills the lions share of the water supply. So this is the only way I've rationalized C. C is basically saying "what if the graduate population exploded such that it's no longer reasonable to say that the military is dependent on dropouts".
I really want help with this. I know I'm wrong with why C is right but otherwise I'm stuck.
This happening to me as well.
this is the first question in a while that ive looked at and just couldnt find the answer. i thought i conquered level 5s but alas .
Ermmmmm 🤓 actually its $5, not $4.50. get your math right JY